5 votos

Derivada de la función de Schur

En su respuesta a http://mathoverflow.net/questions/129854 R. Stanley dice que la derivada parcial (sobre la x[i] relevante) de la función de Schur de una partición lambda de n es igual a la suma de las funciones de Schur sobre las parciones recortadas de lambda:

"Dejemos $s_λ$ denotan una función de Schur y $p1=s1=x1+x2+\dots$ . Entonces, $\frac{\partial\lambda}{\partial p_1}=\sum_\mu s_\mu$ donde el $μ$ se obtienen eliminando una sola casilla de $λ$ ."

Si resuelvo esto para $\lambda = \{3,1\}$ Encuentro para la derivada $12 X[3]+ 108 X[2,1]+ 60 X[1,1,1]$ (sustituyendo $X[i,j,k]$ para los términos $x[1]^i*x[2]^j*x[3]^k$ ) mientras que las particiones recortadas $\{2,1\}$ y $\{3\}$ producir $4 X[3]+ 24 X[2,1] + 12 X[1,1,1]$ .

¿Qué me falta? ¿Puede alguien dar un ejemplo de lo que se pretende?

4voto

GmonC Puntos 114

Stanley utiliza una descripción críptica cuya interpretación es fácil para los iniciados, pero no para los demás. Recordemos que el anillo de funciones simétricas está dotado de un producto interior para el que las funciones de Schur forman una base ortonormal (podemos tomar esto como su definición, aunque también hay otras descripciones). Ahora la operación escrita $\frac\partial{\partial p_1}$ no es más que el adjunto (en el sentido de los mapeos lineales) de la operación de multiplicación por $p_1=s_1=\sum_i x_i$ . Como esa multiplicación se aplica a $s_\lambda$ da la suma de todos los $s_\mu$ donde el diagrama de $~\mu$ se puede obtener añadiendo un cuadrado al de $~\lambda$ su adjunto aplicado a $s_\lambda$ da la suma de todos los $s_\mu$ donde el diagrama de $~\mu$ se puede obtener quitando un cuadrado del de $~\lambda$ .

Una forma de ver esta operación aplicada a cualquier función simétrica $f$ es señalar una de las variables en las que $f$ es una función simétrica (como es simétrica no importa cuál), llama a esa variable $y$ y los demás $x_1,x_2,\ldots$ observando que $f$ es ciertamente simétrica en el $x_1,x_2,\ldots$ escríbalo como un polinomio en $y$ con como coeficientes funciones simétricas en la $x_1,x_2,\ldots$ y luego tomar el coeficiente de la primera potencia $y^1$ de $~y$ como resultado (hemos consumido una variable, pero dada la oferta infinita esto no hace ninguna diferencia; este es el hotel de Hilbert en funcionamiento). En la práctica no se necesitan infinitas variables: se puede escribir $f$ como polinomio simétrico en variables $x_1,\ldots,x_n$ donde $n$ es al menos $\deg f$ y elija $y=x_n$ como "ultimus inter pares", lo que evita la renumeración (el grado desciende convenientemente por $1$ Así que $x_n$ es prescindible).

Si se toma la descripción de $s_\lambda(x_1,\ldots,x_n)$ en términos de tablas semiestructuradas, se puede dividir el conjunto de sus términos según el conjunto de cuadrados que contienen una entrada $~n$ en el retablo. Cada una de las partes de esta partición de términos define un producto $s_\mu(x_1,\ldots,x_{n-1})x_n^d$ , donde $d$ es el número de casillas del conjunto fijo que contienen una entrada $~n$ y el diagrama de $\mu$ consiste en las casillas restantes. De ello se deduce que el coeficiente de $x_n^d$ es la suma de todos los $s_\mu(x_1,\ldots,x_{n-1})$ donde el diagrama de $~\mu$ se obtiene a partir de la de $~\lambda$ eliminando una franja horizontal de tamaño $~d$ . En particular, nuestro operador mapea $s_\lambda$ a la suma de todos los $s_\mu$ cuyo diagrama se obtiene a partir del de $\lambda$ quitando una esquina.

Para ver que el operador coincide con la diferenciación formal respecto a $p_1$ cuando se aplica a un polinomio en $p_1,p_2,\ldots$ interpretadas como funciones simétricas de suma de potencias, se puede demostrar que es una derivación que desaparece en $p_2,p_3,\ldots$ y envía $p_1\mapsto1$ o se puede aplicar directamente nuestra descripción a un producto $p_1^mf$ donde $f$ es un factor en el que el operador desaparece (un monomio en $p_2,p_3,\ldots$ ), utilizando $p_1(x_1,\ldots,x_n)=p_1(x_1,\ldots,x_{n-1})+x_n$ y la expansión de $(p_1(x_1,\ldots,x_{n-1})+x_n)^m$ dando como resultado de la operación $mp_1^{m-1}f$ .

1voto

riza Puntos 170

Lo que entiendo de la respuesta de Stanley en MathOverflow es que:

  • Las funciones de Schur se toman como funciones generadoras en infinitas variables;
  • Para diferenciar wrt $p_1$ , primero hay que escribir $s_\lambda$ como función generadora en la suma de potencias de los polinomios simétricos como $s_\lambda=f(p_1,p_2,\cdots)$ .

Hagamos un ejemplo más sencillo, digamos $\lambda=(2,1)$ . Los cuadros de forma semiestándar $\lambda$ tendrá varias formas posibles; las dividimos en tipos para todos los naturales con $a<b<c$ :

$$\color{blue}{ \begin{matrix} a & b \\ b\end{matrix} \qquad \begin{matrix} a & a \\ b\end{matrix}} \qquad \color{green}{ \begin{matrix} a & b \\ c\end{matrix} \qquad \begin{matrix} a & c \\ b\end{matrix}}$$

Por lo tanto, la función schur es $s_{(2,1)}=\color{blue}{m_{(2,1)}}+\color{green}{2m_{(1,1,1)}}$ . Desde $p_1^3=p_3+3m_{(2,1)}+6m_{(1,1,1)}$ podemos escribir $s_{(2,1)}=(p_1^3-p_3)/3$ Por lo tanto $\partial s_{(2,1)}/\partial p_1=p_1^2=p_2+2m_{(1,1)}$ . Ahora los cuadros de forma semiestándar $\color{purple}{(2)}$ y $\color{teal}{(1,1)}$ son todos de la forma (delimitada con un $|$ )

$$\left. \color{purple}{\begin{matrix}a \\ b\end{matrix}} \quad \right| \quad \color{Teal}{\begin{matrix}a& b\end{matrix} \qquad \begin{matrix}a & a\end{matrix}} $$

Por lo tanto, $\color{purple}{s_{(2)}=m_{(1,1)}}$ y $\color{teal}{s_{(1,1)}=m_{(1,1)}+p_2}$ . Por lo tanto,

$$\frac{\partial s_{(2,1)}}{\partial p_1}=p_2+2m_{(1,1)}=s_{(1,1)}+s_{(2)},$$

exactamente como se dice.

Esto debería ser válido para las funciones de Schur de un número finito de variables; basta con aplicar el método "evaluar $x_l=0$ para todos $l>n$ " a las relaciones $s_\lambda = f(p_1,p_2,\cdots)$ y $\partial f/\partial p_1=\sum s_\mu$ .


Sin embargo, estoy obteniendo un término faltante o extraño en mi propio cálculo que implica $s_{(3,1)}$ . Los tres cuadros de forma semiestándar $(3,1)$ son

$$\begin{matrix}1 & 1 \\ 2 \\ 3\end{matrix} \qquad \begin{matrix}1 & 2 \\ 2 \\ 3\end{matrix} \qquad \begin{matrix}1 & 3 \\ 2 \\ 3\end{matrix}$$

por lo que

$$s_{(3,1)}(x_1,x_2,x_3)=x_1^2x_2x_3+x_1x_2^2x_3+x_1x_2x_3^2=x_1x_2x_3(x_1+x_2+x_3).$$

Consulta de la Fórmulas NG y las recursiones que producen, esto debería ser igual a

$$\frac{p_1^3-3p_1p_2+2p_3}{6}p_1$$

por lo que

$$\frac{\partial s_{(3,1)}}{\partial p_1}=\frac{4p_1^3-6p_1p_2+2p_3}{6}=x_1^2x_2+x_1x_2^2+x_1^2x_3+x_1x_3^2+x_2^2x_3+x_2x_3^2+4x_1x_2x_3.$$

El único retablo semiestándar de la forma $(3)$ es $\begin{matrix}1\\2\\3\end{matrix}$ Por lo tanto $s_{(3)}(x_1,x_2,x_3)=x_1x_2x_3$ .

Los cuadros de forma semiestándar $(2,1)$ son

$$\begin{matrix} 1 & 1 \\ 2\end{matrix} \qquad \begin{matrix} 1 & 1 \\ 3\end{matrix} \qquad \begin{matrix} 1 & 2 \\ 2\end{matrix} \qquad \begin{matrix} 1 & 2 \\ 3\end{matrix} \qquad \begin{matrix} 1 & 3 \\ 2\end{matrix} \qquad \begin{matrix} 1 & 3 \\ 3\end{matrix} \qquad \begin{matrix} 2 & 2 \\ 3\end{matrix} \qquad \begin{matrix} 2 & 3 \\ 3\end{matrix} $$

por lo que

$$s_{(2,1)}(x_1,x_2,x_3)=x_1^2x_2+x_1x_2^2+x_1^2x_3+x_1x_3^2+x_2^2x_3+x_2x_3^2+2x_1x_2x_3.$$

Pero esto indicaría que $\partial s_{(3,1)}/\partial p_1=s_{(2,1)}+s_{(3)}\color{red}{+x_1x_2x_3}$ Así que debo haber cometido un error en alguna parte. Quizás alguien más pueda ver dónde está mi error.

i-Ciencias.com

I-Ciencias es una comunidad de estudiantes y amantes de la ciencia en la que puedes resolver tus problemas y dudas.
Puedes consultar las preguntas de otros usuarios, hacer tus propias preguntas o resolver las de los demás.

Powered by:

X